4
$\begingroup$

The groundbreaking work of Maynard and Tao showed the following fundamental result:

For any integer $m$, there exists a number $k(m)$ such that for any admissible set $H$ of $k$ integers (with $k$ at least $k(m)$), there are infinitely many integers $n$ for which the set $n+H$ contains at least $m$ primes.

My question is about a natural and stronger version. Suppose we want to force primes to appear in different parts of the set simultaneously. More precisely, suppose we are given numbers $r$, and $m_1, m_2, \ldots, m_r$.

Question. Does there exist a constant $C$ (depending on $r$ and all the $m_j$) such that for any admissible set $H$ of $k$ integers (with $k$ at least $C$), and for any way of splitting $H$ into $r$ smaller groups $H_1$, $H_2$,$\ldots$, $H_r$, there are infinitely many integers $n$ with the following property?

For every single group $H_j$ (where $j=1$ to $r$), the set $n+H_j$ contains at least $m_j$ primes.

In other words, can we guarantee that we can find infinitely many $n$ such that each part of the partition contains many primes at the same time?

$\endgroup$
3
  • 5
    $\begingroup$ I doubt it, since this would imply the twin prime conjecture: take $r=2$ and $m_1=2$ and take a very large admissible set $H$ containing $0$ and $2$, and $H_1=\{0,2\}$. You should probably add a condition like each $H_j$ is sufficiently large depending on $m_j$. $\endgroup$ Commented Nov 18 at 10:08
  • 2
    $\begingroup$ This is obviously false if $m_j<|H_j|$, but as Thomas says, you probably want to ask for $H_j$ large enough relative to $m_j$. I think an equivalent formulation would be: for any $m,r$ there some $C$ such that given $r$ admissible tuples $H_i$ of size $k$, there are infinitely many $n$ such that $n+H_i$ contains $m$ primes for each $i$. That is, we ask for Maynard-Tao for a number of admissible tuples simultaneously. $\endgroup$ Commented Nov 18 at 11:38
  • $\begingroup$ Thank you, Thomas Bloom and Wojowu; $H_j$ needs to be sufficiently large relative to $m_j$. $\endgroup$ Commented Nov 19 at 0:22

1 Answer 1

5
$\begingroup$

I don't think this can be done even for $H_j$ arbitrarily large in terms of $m_j$. The method of Maynard's work, and prior work in the same direction, proceeds roughly by finding a set $S$ of $n$, expressed in terms of arithmetic progressions, such that $n+h$ has no small prime factors for $n \in S$ and $h\in H$, then applying results counting primes in arithmetic progressions to show that many numbers $n+h$ must be prime, which implies by a pigeonhole argument that for some $n$ we must have $n+h$ prime for many $h$.

The problem is that even if for each $j$ and each $h \in H_j$ we know that $n+h$ is prime for many $n \in S$, the pigeonhole doesn't tell us that we simultaneously have $n+h_1$ and $n+h_2$ prime for $h_1 \in H_1$ and $h_2\in H_2$. Rather, we would need some control on the number of $n\in S$ with $n+h_1$ prime and $n+h_2$ prime, which would be stronger than the twin prime conjecture instead of merely a variant of the Bombieri-Vinogradov theorem.

So it seems to me that, if this could be done, it would require a completely new method rather than a small variation on existing techniques.

$\endgroup$
1
  • $\begingroup$ Thank you, Will Sawin $\endgroup$ Commented Nov 19 at 0:23

You must log in to answer this question.

Start asking to get answers

Find the answer to your question by asking.

Ask question

Explore related questions

See similar questions with these tags.